A simple sketch of how realism became unpopular

post by Rob Bensinger (RobbBB) · 2019-10-11T22:25:36.357Z · LW · GW · 55 comments

[Epistemic status: Sharing current impressions in a quick, simplified way in case others have details to add or have a more illuminating account. Medium-confidence that this is one of the most important parts of the story.]


Here's my current sense of how we ended up in this weird world where:


Background context:

1. The ancient Greeks wrote down a whole lot of arguments. In many cases, we're missing enough textual fragments or context that we don't really know why they were arguing — what exact propositions were in dispute, or what the stakes were.

2. In any case, most of this is screened off by the fact that Europe's memetic winners were Christianity plus normal unphilosophical beliefs like "the sky is, in fact, blue".

3. Then, in 1521, the Protestant Reformation began.

4. In 1562, the Catholics found a giant list of arguments against everything by the minor Greek skeptic Sextus Empiricus, got very excited, and immediately weaponized them to show that the Protestant arguments fail (because all arguments fail).

5. These soon spread and became a sensation, and not just for being a useful superweapon. Plenty of intellectuals were earnest humanists used to taking arguments at face value, and found Sextus' arguments genuinely upsetting and fascinating.


I trace continental thinkers' "everything is subjective/relative" arguments back to a single 1710 error in George Berkeley:

[...] I am content to put the whole upon this Issue; if you can but conceive it possible for one extended moveable Substance, or in general, for any one Idea or any thing like an Idea, to exist otherwise than in a Mind perceiving it, I shall readily give up the Cause[....]
But say you, surely there is nothing easier than to imagine Trees, for instance, in a Park, or Books existing in a Closet, and no Body by to perceive them. I answer, you may so, there is no difficulty in it: But what is all this, I beseech you, more than framing in your Mind certain Ideas which you call Books and Trees, and the same time omitting to frame the Idea of any one that may perceive them? But do not you your self perceive or think of them all the while? This therefore is nothing to the purpose: It only shews you have the Power of imagining or forming Ideas in your Mind; but it doth not shew that you can conceive it possible, the Objects of your Thought may exist without the Mind: To make out this, it is necessary that you conceive them existing unconceived or unthought of, which is a manifest Repugnancy.

If I can imagine a tree that exists outside of any mind, then I can imagine a tree that is not being imagined. But "an imagined X that is not being imagined" is a contradiction. Therefore everything I can imagine or conceive of must be a mental object.

Berkeley ran with this argument to claim that there could be no unexperienced objects, therefore everything must exist in some mind — if nothing else, the mind of God.

The error here is mixing up what falls inside vs. outside of quotation marks. "I'm conceiving of a not-conceivable object" is a formal contradiction, but "I'm conceiving of the concept 'a not-conceivable object'" isn't, and human brains and natural language make it easy to mix up levels like those.

(I can immediately think of another major milestone in the history of European thought, Anselm's ontological argument for God, that shows the same brain bug.)

Berkeley's view was able to find fertile soil in an environment rife with non-naturalism, skeptical arguments, and competition between epistemic criteria and authorities. Via Kant and Kant's successors (Hegel chief among them), he successfully convinced the main current of 19th-century European philosophy to treat the idea of a "mind-independent world" as something ineffable or mysterious, and to treat experiences or perspectives as fundamental.

(Edit: G.E. Moore seems to think [LW(p) · GW(p)] that everyone in the 19th century was making an error along these lines, but I now suspect [LW(p) · GW(p)] Kant himself wasn't making this mistake; I think his main error was trying too hard to defeat skepticism.

I also don't think Berkeley's writing would have been sufficient to confuse Europe on its own; it's too lucid and well-articulated. The transition to Kantian and Hegelian versions of these arguments is important because they were much more elaborate and poorly expressed, requiring a lot of intellectual effort in order to spot the inconsistencies.)

My unscholarly surface impression of the turn of the 20th century is that these memes ("the territory is fundamentally mysterious" and "maps are sort of magical and cosmically important") allowed a lot of mysticism and weird metaphysics to creep into intellectual life, but that ideas like those are actually hard to justify in dry academic prose, such that the more memetically fit descendants of idealism in the 20th century ended up being quietist ("let's just run experiments and not talk about all this weird 'world' stuff") or instrumentalist / phenomenalist / skeptic / relativist ("you can't know 'world' stuff, so let's retreat to just discussing impressions; and maybe you can't even know those, so really what's left is power struggles").

Today, the pendulum has long since swung back again in most areas of intellectual life, perhaps because we've more solidly settled around our new central authority (science) and the threats to centralized epistemic authority (religious and philosophical controversy) are more distant memories. Metaphysics and weird arguments are fashionable again in analytic philosophy; behaviorism is long-dead in psychology; and quietism, non-realism, and non-naturalism at least no longer dominate the discussion in QM, though a lot of Copenhagen slogans remain popular.


The above is a very simple picture featuring uneven scholarship, and history tends to be messier than all that. (Ideas get independently rediscovered, movements go one step forward only to retreat two steps back, etc.) Also, I'm not claiming that everyone endorsed the master argument as stated, just that the master argument happened to shift intellectual fashions in this direction in a durable way.

55 comments

Comments sorted by top scores.

comment by Vanessa Kosoy (vanessa-kosoy) · 2019-10-12T12:29:49.433Z · LW(p) · GW(p)

This post was educational, however, I want to push back against the implicit criticism of instrumentalism and the Copenhagen interpretation. The metaphilosophical position I will use here is: to solve a philosophical question, we need to rephrase it as a question about AI design (AFAIK the full credit for this approach goes to Yudkowsky). So, suppose I am building an AI. Should the AI's model of the world be (i) "about things that exist independently of the AI" or (ii) "about the subjective perceptions of the AI"? This depends on what kind of reward function I want my AI to have.

The standard (I call it "perceptible") type of reward function in reinforcement learning only depends on the history of actions and observations. For such an AI the answer is (ii): it is important the AI will correctly predict the consequences of its actions, but there is no significance whatsoever that the AI's models can be interpreted as (i). Yes, these models will still have intrinsic variables corresponding to "unobserved objects" in some sense, but there is no reason for these variables to always have an unambiguous "realist" interpretation.

Now suppose that the AI is actually designed to care about particular objects outside itself. Specifically, assume the AI uses an instrumental (or semi-instrumental) reward function [AF · GW]. Such a function might be specified (or partially specified) using some particular ontology. Then, the AI is modeling the world as containing certain unobserved objects, at least approximately. The answer is now in between (i) and (ii). The AI's models are about subjective perceptions of the AIs and also about some particular type of things that exist independently of the AI, namely, those things that are normatively important to it. Coming back from AIs to humans, we may conclude that, what really makes sense is our subjective perceptions + those external objects that we actually care about (e.g. other humans). But, quarks only "exist" in the sense that they are components in a useful model we created.

What does it tell us about the interpretation of quantum mechanics? Once again, consider an AI trying to discover quantum mechanics. From the AI's perspective, what it's looking for is a function from observation histories to distributions over the next observation. How can we construct such a function from the formalism of QM? Obviously, using Copenhagen: each observation is a measurement that causes the wavefunction of the environment to collapse. "But," the fans of MWI will object "what about all those other Everett branches? They are still out there, right? They don't actually vanish?!" The point is, from the AI's perspective the question is meaningless. The AI's design assumes the AI can record everything it observes in its memories, therefore, once an observation is made, those Everett branches will never meet again in the AI's lifetime. "But," another objection may go "what if someone tempers with the memories of the AI in a way that allows quantum interference between the branches? Sure it is completely impractical for humans, but it is theoretically possible, and might even be practically possible for an AI running on a quantum computer." Alright, but tempering with the memory of the agent is effectively destroying the agent: it invalidates the fundamental assumptions of its reasoning algorithm, and any reasoning algorithm must make some fundamental assumptions of that sort. (The agent might still accurately account for the possibility of its own destruction (see "The Death of the Agent and Kamikaze Strategies" [AF · GW]), but probably only with the help of external knowledge.)

Replies from: RobbBB
comment by Rob Bensinger (RobbBB) · 2019-10-12T22:05:24.268Z · LW(p) · GW(p)
This post was educational, however, I want to push back against the implicit criticism of instrumentalism and the Copenhagen interpretation. The metaphilosophical position I will use here is: to solve a philosophical question, we need to rephrase it as a question about AI design

Maybe the problem is that I'm not sufficiently convinced that there's a philosophical question here. Sometimes philosophers (and even physicists) argue about things that aren't open questions. "Do refrigerators exist, or only mental models of refrigerators?" sounds like a straightforward, testable empirical question to me, with all the evidence favoring "refrigerators exist".

I predict I'm missing an implicit premise explaining why "I don't currently understand where the Born rule comes from" is a bigger problem for realism than "I don't currently understand how my refrigerator works", or some other case where realism makes things unnecessarily hard/confusing, like infinite ethics or anthropics or somesuch.

Replies from: vanessa-kosoy
comment by Vanessa Kosoy (vanessa-kosoy) · 2019-10-13T19:44:21.849Z · LW(p) · GW(p)

Let's unpack what it means to say that "refrigerators exist". From my (instrumentalist) perspective, it means that (i) I have a predictive model of my perception, to which I assign high credence, and in which for each state of the environment I can say which refrigerators exist (where "refrigerator" is just some symbol that makes sense inside the model) and (ii) according to my belief, the current state of the environment contains at least one refrigerator with high probability.

My claim is not that quantum mechanics proves realism is wrong. My claim is that instrumentalism is the correct metaphysics regardless, and once you accept that, the Copenhagen interpretation seems quite satisfactory. Although it is also true that if you try interpreting quantum mechanics according to sufficiently strong realist desiderata you run into impossibility results like the Kochen-Specker theorem and the violation of Bell's inequality.

Replies from: RobbBB
comment by Rob Bensinger (RobbBB) · 2019-10-13T20:33:18.068Z · LW(p) · GW(p)
My claim is that instrumentalism is the correct metaphysics regardless

What does it mean for instrumentalism to be the correct metaphysics? Normally, I'd interpret "the correct metaphysics" as saying something basic about reality or the universe. (Or, if you're an instrumentalist and you say "X is the correct metaphysics", I'd assume you were saying "it's useful to have a model that treats X as a basic fact about reality or the universe", which also doesn't make sense to me if X is "instrumentalism".)

Although it is also true that if you try interpreting quantum mechanics according to sufficiently strong realist desiderata

Well, sufficiently specific realist desiderata. Adding hidden variables to QM doesn't make the theory any more realist, the way we're using "realist" here.

Replies from: vanessa-kosoy
comment by Vanessa Kosoy (vanessa-kosoy) · 2019-10-14T12:06:02.236Z · LW(p) · GW(p)

What does it mean for instrumentalism to be the correct metaphysics? Normally, I'd interpret "the correct metaphysics" as saying something basic about reality or the universe. (Or, if you're an instrumentalist and you say "X is the correct metaphysics", I'd assume you were saying "it's useful to have a model that treats X as a basic fact about reality or the universe", which also doesn't make sense to me if X is "instrumentalism".)

Like I said before, it means that instrumentalism is the point of view that is the most useful for designing AI or answering questions about AI. According to the "Yudkowskian computationalist" metaphilosophical view, this also makes it the most useful for rationality in general.

Adding hidden variables to QM doesn't make the theory any more realist, the way we're using "realist" here.

I imagined "realist" to mean something like "the universe can be described in a way independent of the choice of observer, and the perceptions of any given observer can be decoded from the history of the universe in this description, s.t. different observers have compatible observations". Adding hidden variables does make QM more realist in this sense, for example the de Broglie-Bohm interpretation is realist (but it only makes sense if you assume all observer perceptions can be reduced to configuration variables, which seems false and disqualifies it). MWI fails to be entirely realist because you have to either make the decoding of observer perceptions stochastic (and thereby admit that your description of the universe is incomplete) or to postulate, for each "normal" observer Alice, a huge ensemble of different observers that correspond to versions of Alice in different Everett branches (and thereby lose the physical meaning of quantum probabilities and with it essentially all the predictive power of the theory).

Replies from: TAG
comment by TAG · 2019-10-17T10:20:40.772Z · LW(p) · GW(p)

Like I said before, it means that instrumentalism is the point of view that is the most useful for designing AI or answering questions about AI. According to the “Yudkowskian computationalist” metaphilosophical view, this also makes it the most useful for rationality in general.

Except that if you are the kind of rationalist who cares about what is really real, you should reject instrumentalism immediately.

comment by Dustin · 2019-10-12T00:27:26.808Z · LW(p) · GW(p)
The error here is mixing up what falls inside vs. outside of quotation marks. "I'm conceiving of a not-conceivable object" is a formal contradiction, but "I'm conceiving of the concept 'a not-conceivable object'" isn't, and human brains and natural language make it easy to mix up levels like those.

I immediately saw this mistake *while reading the text of the mistake.*

So, now I'm confused. Am I master ninja of a philosopher? Are you misrepresenting the level of people's confusion about this? Have I been arguing on the internet for 3 decades and thus I'm just hypersensitive to language/text based mistakes? Are we both wrong that this is a mistake?

Replies from: clone of saturn, Gurkenglas, RobbBB, TAG
comment by clone of saturn · 2019-10-12T02:12:38.829Z · LW(p) · GW(p)

It's relatively easy to be a master ninja of philosophy relative to anyone in 1710, because you've had a chance to crib from the work of all the smartest people who've lived since then.

Replies from: Dustin
comment by Dustin · 2019-10-12T15:50:07.176Z · LW(p) · GW(p)

I can see this being true, but I'm not entirely convinced.

I have no background in philosophy. I don't read philosophy other than occasionally dipping into LW.

Of course, there exists the possibility that occasional dipping into LW has been enough, or that the necessary mental rigor has just seeped into the general populace over the intervening few hundred years.

Also, I'm not sure "anyone in 1710" is the right comparison. More like "people thinking about philosophy in 1710".

Of course, that is likely what you meant, but I think the less precise wording you used makes your argument a lot more convincing so I think it's important to point out the distinction.

To be clear, I'm not arguing that actually I am a ninja of philsophy. I'm just saying that your point doesn't necessarily make me less confused.

Replies from: romeostevensit
comment by romeostevensit · 2019-10-12T18:28:31.572Z · LW(p) · GW(p)

It's also that a lot of philosophy, including famous works, is just really bad.

Replies from: SaidAchmiz
comment by Gurkenglas · 2019-10-12T01:37:55.176Z · LW(p) · GW(p)

It's rather obvious if you've done programming or studied provability or read the sequences. The lesswrong crowd isn't a good sample for testing the strength of this trap.

comment by Rob Bensinger (RobbBB) · 2019-10-12T04:17:30.587Z · LW(p) · GW(p)

Re "was Berkeley making such an obvious mistake?", I think this is historians' majority view, but multiple people have tried to come up with more reasonable versions of the argument; see Gallois (1974) and Downing (2011). Note that Berkeley makes the same argument in dialogue form here (starts at "How say you, Hylas, can you see a thing which is at the same time unseen?"), so you can check if you find that version more tenable.

The Bloomsbury Companion to Berkeley says:

[This passage] can be interpreted as making a straightforward howler, arguing that because whenever you think of something it is being thought of and anything being thought of is, ipso facto, 'in the mind' then you cannot think of something that is not in the mind. According to Russell, this was a keystone for idealism and it involves a simple mistake.

"Berkeley's view . . . seems to depend for its plausibility upon confusing the thing apprehended with the act of apprehension. Either of these might be called an 'idea'; probably either would have been called an idea by Berkeley. The act is undoubtedly in the mind; hence, when we are thinking of the act, we readily assent to the view that ideas must be in the mind. Then, forgetting that this was only true when ideas were taken as acts of apprehension, we transfer the proposition that 'ideas are in the mind' to ideas in the other sense, i.e. to the things apprehended by our acts of apprehension. Thus, by an unconscious equivocation, we arrive at the conclusion that whatever we apprehend must be in our minds."
1912/1967,22

Russell's criticism is in line with Moore's famous 'The Refutation of Idealism' (1903), where he argues that if one recognizes the act-object distinction within conscious states, one can see that the object is independent of the act. This 'discovery', together with the development of a formal logic for relations, was the cornerstone of the rejection of 'British idealism'. If objects can be conceived of as independent of conscious thought, and if it is consistent to think of them as in actually related to each other, the mentalistic holism that was contemporary idealism is demolished.

That said, I put a lot of weight on Allen Wood's view as a leading Kant scholar, and revisiting his book Kant, he doesn't think Kant accepted the master argument (p. 69). David (2015) asserts a link, but it looks tenuous to me.

Kant's earliest interpreters took him to be saying "trees, oceans, etc. all just exist in your head and have nothing in common with the mysterious ineffable things-in-themselves", and Kant definitely talks like that a great deal, but he also says a lot that contradicts that view. Wood thinks Kant was just really confused and fuzzy about his own view, and didn't have a consistent model here (pp. 63-71).

My new pet theory is that Kant was being pulled in one direction by "wanting to make things as subjective as possible so he can claim more epistemic immediacy and therefore more immunity to skeptical arguments", and in the opposite direction by "not wanting to sound like a crazy person like Berkeley", so we get inconsistencies.

I don't know who, if anyone, noted the obvious fallacy in Berkeley's master argument prior to Russell in 1912, and Russell seems to think the argument was central to idealism's appeal. Regardless, my new view is: philosophy mainly ended up going down an idealist cul-de-sac because Kant shared Berkeley's "try to find ways to treat more things as subjective" approach to defeating skepticism. (Possibly without realizing it; Stang (2016) suggests Kant was pretty confused about what Berkeley believed.) Then Kant and Hegel built sufficiently dense, mysterious, and complicated intellectual edifices that it was easy for them to confuse themselves and others, while still being brilliant, innovative, and internally consistent enough to attract a lot of followers.

Replies from: ESRogs, TAG
comment by ESRogs · 2019-10-12T08:15:35.045Z · LW(p) · GW(p)
I don't know who, if anyone, noted the obvious fallacy in Berkeley's master argument prior to Russell in 1912

Not even Moore in 1903?

Russell's criticism is in line with Moore's famous 'The Refutation of Idealism' (1903), where he argues that if one recognizes the act-object distinction within conscious states, one can see that the object is independent of the act.

Isn't that the same argument Russell was making?

Replies from: RobbBB
comment by Rob Bensinger (RobbBB) · 2019-10-12T21:19:42.796Z · LW(p) · GW(p)
Isn't that the same argument Russell was making?

They're... similar? I find Russell a lot clearer on this point:

We might state the argument by which they support their view in some such way as this: 'Whatever can be thought of is an idea in the mind of the person thinking of it; therefore nothing can be thought of except ideas in minds; therefore anything else is inconceivable, and what is inconceivable cannot exist.'
Such an argument, in my opinion, is fallacious; and of course those who advance it do not put it so shortly or so crudely. But whether valid or not, the argument has been very widely advanced in one form or another; and very many philosophers, perhaps a majority, have held that there is nothing real except minds and their ideas.

I find Moore's way of discussing the issue weirder. Moore is definitely making an argument from 'a thought's object is different from the thought itself' to 'idealism is false', but his argument seems to involve weird steps like 'our experiences don't have contents' (rather than the expected 'the content of our experience is different from its referent'):

What I wish to point out is (1) that we have no reason for supposing that there are such things as mental images at all -- for supposing that blue is part of the content of the sensation of blue, and (2) that even if there are mental images, no mental image and no sensation or idea is merely a thing of this kind; that 'blue', even if it is part of the content of the image or sensation or idea of blue, is always also related to it in quite another way, and that this other relation, omitted in the traditional analysis, is the only one which makes the sensation of blue a mental fact at all. [...]
To have in your mind 'knowledge' of blue, is not to have in your mind a 'thing' or 'image' of which blue is the content. To be aware of the sensation of blue is not to be aware of a mental image - of a 'thing', of which 'blue' and some other element are constituent parts in the same sense in which blue and glass are constituents of a blue bead. It is to be aware of an awareness of blue; awareness being used, in both cases, in exactly the same sense. This element, we have seen, is certainly neglected by the 'content' theory: that theory entirely fails to express the fact that there is, in the sensation of blue, this unique relation between blue and the other constituent.

Baldwin (2004) confirms that this line of reasoning, plus Moore's attempt to resist skeptical hypotheses, led Moore in a very confused direction:

But what is the relationship between sense-data [i.e., the thingies we're directly conscious of] and physical objects? Moore took it that there are three serious candidates to be considered: (i) an indirect realist position, according to which sense-data are non-physical but somehow produced by interactions between physical objects and our senses; (ii) the phenomenalist position, according to which our conception of physical objects is merely one which expresses observed and anticipated uniformities among the sense-data we apprehend; (iii) a direct realist position, according to which sense-data are parts of physical objects — so that, for example, visual sense-data are visible parts of the surfaces of physical objects.
The indirect realist position is that to which he was initially drawn; but he could see that it leaves our beliefs about the physical world exposed to skeptical doubt, since it implies that the observations which constitute evidence for these beliefs concern only the properties of non-physical sense-data, and there is no obvious way for us to obtain further evidence to support a hypothesis about the properties of the physical world and its relationship to our sense-data.
This argument is reminiscent of Berkeley's critique of Locke, and Moore therefore considered carefully Berkeley's phenomenalist alternative. Moore's initial response to this position was that the implied conception of the physical world was just too ‘pickwickian’ to be believable. This may be felt to be too intuitive, like Dr. Johnson's famous objection to Berkeley; but Moore could also see that there were substantive objections to the phenomenalist position, such as the fact that our normal ways of identifying and anticipating significant uniformities among our sense-data draw on our beliefs about our location in physical space and the state of our physical sense-organs, neither of which are available to the consistent phenomenalist.
So far Moore's dialectic is familiar. What is unfamiliar is his direct realist position, according to which sense-data are physical. This position avoids the problems so far encountered, but in order to accommodate false appearances Moore has to allow that sense-data may lack the properties which we apprehend them as having. It may be felt that in so far as sense-data are objects at all, this is inevitable; but Moore now needs to provide an account of the apparent properties of sense-data and it is not clear how he can do this without going back on the initial motivation for the sense-datum theory by construing these apparent properties as properties of our experiences. But what in fact turns Moore against this direct realist position is the difficulty he thinks it leads to concerning the treatment of hallucinations. In such cases, Moore holds, any sense-data we apprehend are not parts of a physical object; so direct realism cannot apply to them, and yet there is no reason to hold that they are intrinsically different from the sense-data which we apprehend in normal experience. This last point might well be disputed, and at one point Moore himself considers the possibility of a distinction between ‘subjective’ and ‘objective’ sense-data; but once one has introduced sense-data in the first place as the primary objects of experience it is not going to be easy to make a distinction here without assuming more about experience than Moore at any rate would have wanted to concede.
Moore wrote more extensively about perception than about any other topic. In these writings he moves between the three alternatives set out here without coming to any firm conclusion.
Replies from: RobbBB, RobbBB
comment by Rob Bensinger (RobbBB) · 2019-10-12T21:43:50.501Z · LW(p) · GW(p)

I should note that Russell was also led in some pretty weird directions by the desire to resist skeptical arguments:

What distinguishes neutral monism from its monistic rivals is the claim that the intrinsic nature of ultimate reality is neither mental nor physical. [...] Following a series of critical engagements with neutral monism (see especially Russell 1914a,b), Russell adopted it in Russell 1919 and remained a neutral monist for the rest of his long career: “I am conscious of no major change in my opinions since the adoption of neutral monism” is what he says in an interview from 1964 (Eames 1969: 108). [...]
For an entity to be neutral is to “have neither the hardness and indestructibility of matter, nor the reference to objects which is supposed to characterize the mind” (Russell 1921: 36; cf. 124). Russell never suspected sensations of being material (in this sense). That sensations are mental (in this sense)—that they consist of a mental act of sensing directed at a non-mental object—was, however, a pivotal part of his earlier view. But then his views changed:
"I formerly believed that my own inspection showed me the distinction between a noise [the object] and my hearing of a noise [the act of sensing], and I am now convinced that it shows me no such thing, and never did." (Russell 1918b: 255)

Also, insofar as Moore and Russell are gesturing at similar issues, Moore's paper provides some support for the claim that master-argument-ish reasoning was central to the idealism and rested on a simple error that was concealed by motivated reasoning and obfuscatory language, and that no one noticed (or successfully popularized) the error prior to Moore/Russell:

I am suggesting that the Idealist maintains that object and subject are necessarily connected, mainly because he fails to see that they are distinct, that they are two, at all. When he thinks of 'yellow' and when he thinks of the 'sensation of yellow', he fails to see that there is anything whatever in the latter which is not in the former. [...]
But I am well aware that there are many Idealists who would repel it as an utterly unfounded charge that they fail to distinguish between a sensation or idea and what I will call its object. And there are, I admit, many who not only imply, as we all do, that green is distinct from the sensation of green, but expressly insist upon the distinction as an important part of their system. They would perhaps only assert that the two form an inseparable unity.
But I wish to point out that many, who use this phrase, and who do admit the distinction, are not thereby absolved from the charge that they deny it. For there is a certain doctrine, very prevalent among philosophers nowadays, which by a very simple reduction may be seen to assert that two distinct things both are and are not distinct. A distinction is asserted; but it is also asserted that the things distinguished form an 'organic unity'. But, forming such a unity, it is held, each would not be what it is apart from its relation to the other. Hence to consider either by itself is to make an illegitimate abstraction.
The recognition that there are 'organic unities' and 'illegitimate abstractions' in this sense is regarded as one of the chief conquests of modern philosophy. But what is the sense attached to these terms? An abstraction is illegitimate, when and only when we attempt to assert of a part - of something abstracted - that which is true only of the whole to which it belongs: and it may perhaps be useful to point out that this should not be done. But the application actually made of this principle, and what perhaps would be expressly acknowledged as its meaning, is something much the reverse of useful. The principle is used to assert that certain abstractions are in all cases illegitimate; that whenever you try to assert anything whatever of that which is part of an organic whole, what you assert can only be true of the whole. And this principle, so far from being a useful truth, is necessarily false. For if the whole can, nay must, be substituted for the part in all propositions and for all purposes, this can only be because the whole is absolutely identical with the part.
When, therefore, we are told that green and the sensation of green are certainly distinct but yet are not separable, or that it is an illegitimate abstraction to consider the one apart from the other, what these provisos are used to assert is, that though the two things are distinct yet you not only can but must treat them as if they were not. Many philosophers, therefore, when they admit a distinction, yet (following the lead of Hegel) boldly assert their right, in a slightly more obscure form of words, also to deny it. The principle of organic unities, like that of combined analysis and synthesis, is mainly used to defend the practice of holding both of two contradictory propositions, wherever this may seem convenient.
In this, as in other matters, Hegel's main service to philosophy has consisted in giving a name to and erecting into a principle, a type of fallacy to which experience had shown philosophers, along with the rest of mankind, to be addicted. No wonder that he has followers and admirers. [...]
And at this point I need not conceal my opinion that no philosopher has ever yet succeeded in avoiding this self-contradictory error: that the most striking results both of Idealism and of Agnosticism are only obtained by identifying blue with the sensation of blue: that esse ["existing"] is held to be percipi ["being perceived"], solely because what is experienced is held to be identical with the experience of it. That Berkeley and Mill committed this error will, perhaps, be granted: that modern Idealists make it will, I hope, appear more probable later.

This updates me partway back toward the original claim I made (that Berkeley's master argument was causally important for the rise of idealism and its 20th-century successors).

comment by Rob Bensinger (RobbBB) · 2019-10-19T19:07:08.136Z · LW(p) · GW(p)

Also interesting: "insistence that we be immune to skeptical arguments" and "fascination with the idea of representation/intentionality/'aboutness'" seems to have led the continental philosophers in similar directions, as in Sartre's "Intentionality: A Fundamental Idea of Husserl’s Phenomenology." But that intellectual tradition had less realism, instrumentalism, and love-of-science in its DNA, so there was less resistance to sliding toward an "everything is sort of subjective" position.

comment by TAG · 2019-10-26T13:39:46.371Z · LW(p) · GW(p)

Wood thinks Kant was just really confused and fuzzy about his own view, and didn’t have a consistent model here (pp. 63-71).

Maybe confused, maybe nuanced, but certainly not backing some clear but extreme view.

Replies from: RobbBB
comment by Rob Bensinger (RobbBB) · 2019-10-26T15:52:17.254Z · LW(p) · GW(p)

I'd say: definitely nuanced. Definitely very inconsistent on this point. Not consistently asserting an extreme metaphysical view like "the true, mind-independent world is incomprehensibly different from the world we experience", though seeming to flirt with this view (or framing?) constantly, to the extent that all his contemporaries did think he had a view at least this weird. Mainly guilty of (a) muddled and poorly-articulated thoughts and (b) approaching epistemology with the wrong goals and methods.

comment by TAG · 2019-10-15T18:21:31.047Z · LW(p) · GW(p)

Berkely's argument caused a fair amount of incredulity at the time. Samuel Johnon's Argumentum ad Lapidum was intended as a reponse.

BTW, I notice that a lot of people here are persuaded by Aumann's Agreement Theorem, which is every bit as flawed in my view.

Replies from: RobbBB
comment by Rob Bensinger (RobbBB) · 2019-10-18T14:22:49.771Z · LW(p) · GW(p)
Berkely's argument caused a fair amount of incredulity at the time. Samuel Johnon's Argumentum ad Lapidum was intended as a reponse.

This seems like incredulity at his conclusion, rather than at his argument. Do you know of good criticisms of the master argument from the time? (Note it wasn't given a standard name until the 1970s.)

To be clear, I think Berkeley was near-universally rejected at the time, because his conclusion ('there's no material world') was so wild. Most people also didn't understand what Berkeley was saying, even though he was pretty clear about it (see: Kant's misunderstanding; and the above fallacious counterargument, assuming it wasn't just a logically rude joke on Johnson's part).

But I don't update positively about people for rejecting silly-sounding conclusions just based on how silly they sound. The same "that sounds silly" heuristic that helps you reject Berkeley's argument (when it's fringe and 'wears its absurdity on its sleeve') helps you accept 19th-century idealists' versions of the argument (when it's respectable and framed as the modern/scientific/practical/educated/consensus view on the issue).

BTW, I notice that a lot of people here are persuaded by Aumann's Agreement Theorem, which is every bit as flawed in my view.

Flawed how?

Replies from: RobbBB, TAG, TAG
comment by Rob Bensinger (RobbBB) · 2019-10-18T15:56:48.174Z · LW(p) · GW(p)
The same "that sounds silly" heuristic that helps you reject Berkeley's argument (when it's fringe and 'wears its absurdity on its sleeve') helps you accept 19th-century idealists' versions of the argument (when it's respectable and framed as the modern/scientific/practical/educated/consensus view on the issue).

I should also emphasize that Berkeley's idealism is very different from (e.g.) Hegel's idealism. "Idealism" comes in enough different forms that it's probably more useful for referring to a historical phenomenon than a particular ideology. (Fortunately, the former is the topic I'm interested in here.)

comment by TAG · 2019-10-31T11:47:49.992Z · LW(p) · GW(p)

Aumanns theorem assumes there is no ambiguity or disagreement about what counts as evidence or information in the first place. In the real world, that is the major barrier to achieving agreement.

comment by TAG · 2019-10-29T17:59:19.256Z · LW(p) · GW(p)

The same “that sounds silly” heuristic that helps you reject Berkeley’s argument (when it’s fringe and ‘wears its absurdity on its sleeve’) helps you accept 19th-century idealists’ versions of the argument (when it’s respectable and framed as the modern/scientific/practical/educated/consensus view on the issue).

Well, maybe that is right. You haven't really argued against sophisticated idealism, or noticed that scientific realism has dumb forms.

comment by redxaxder · 2019-10-18T23:57:20.667Z · LW(p) · GW(p)
a lot of 20th-century psychologists made a habit of saying things like 'minds don't exist, only behaviors';

It seems like you might be referring to Eliminativism. If you are, this isn't a fair account of it.

Eliminativism isn't opposed to realism. It's just a rejection of the assumption that the labels we apply to people's mental states (wants, believes, loves, etc) are a reflection of the underlying reality. People have been thinking about minds in terms of those concepts for a really long time, but nobody had bothered to sit down and demonstrate that these are an accurate model.

From wiki:

Proponents of this view, such as B.F. Skinner, often made parallels to previous superseded scientific theories (such as that of the four humours, the phlogiston theory of combustion, and the vital force theory of life) that have all been successfully eliminated in attempting to establish their thesis about the nature of the mental. In these cases, science has not produced more detailed versions or reductions of these theories, but rejected them altogether as obsolete. Radical behaviorists, such as Skinner, argued that folk psychology is already obsolete and should be replaced by descriptions of histories of reinforcement and punishment.
Replies from: RobbBB
comment by Rob Bensinger (RobbBB) · 2019-10-19T17:41:40.282Z · LW(p) · GW(p)

Upvoted! My discussion of a bunch of these things above is very breezy, and I approve of replacing the vague claims with more specific historical ones. To clarify, here are four things I'm not criticizing:

  • 1. Eliminativism about particular mental states, of the form 'we used to think that this psychological term (e.g., "belief") mapped reasonably well onto reality, but now we understand the brain well enough to see it's really doing [description] instead, and our previous term is a misleading way of gesturing at this (or any other) mental process.'

I'm an eliminativist (or better, an illusionist) about subjectivity and phenomenal consciousness myself. (Though I think the arguments favoring that view are complicated and non-obvious, and there's no remotely intellectually satisfying illusionist account of what the things we call "conscious" really consist in.)

  • 2. In cases where the evidence for an eliminativist hypothesis isn't strong, the practice of having some research communities evaluate eliminativism or try eliminativism out and see if it leads in any productive directions. Importantly, a community doing this should treat the eliminativist view as an interesting hypothesis or an exploratory research program, not in any way as settled science (or pre-scientific axiom!).
  • 3. Demanding evidence for claims, and being relatively skeptical of varieties of evidence that have a poor track record, even if they "feel compelling".
  • 4. Demanding that high-level terms be in principle reducible to lower-level physical terms (given our justified confidence in physicalism and reductionism).

In the case of psychology, I am criticizing (and claiming really happened, though I agree that these views weren't as universal, unquestioned, and extreme as is sometimes suggested):

  • Skinner's and other behaviorists' greedy reductionism; i.e., their tendency to act like they'd reduced or explained more than they actually had [LW · GW]. Scientists should go out of their way to emphasize the limitations and holes in their current models, and be very careful (and fully explicit about why they believe this) when it comes to claims of the form 'we can explain literally everything in [domain] using only [method].'
  • Rushing to achieve closure, dismiss open questions, forbid any expressions of confusion [? · GW] or uncertainty, and treat blank parts of your map as though they must correspond to a blank (or unimportant) territory. Quoting Watson (1928):
With the advent of behaviorism in 1913 the mind-body problem disappeared — not because ostrich-like its devotees hid their heads in the sand but because they would take no account of phenomena which they could not observe. The behaviorist finds no mind in his laboratory — sees it nowhere in his subjects. Would he not be unscientific if he lingered by the wayside and idly speculated upon it; just as unscientific as the biologists would be if they lingered over the contemplation of entelechies, engrams and the like. Their world and the world of the behaviorist are filled with facts — with data which can be accumulated and verified by observation — with phenomena which can be predicted and controlled.
If the behaviorists are right in their contention that there is no observable mind-body problem and no observable separate entity called mind — then there can be no such thing as consciousness and its subdivision. Freud's concept borrowed from somatic pathology breaks down. There can be no festering spot in the substratum of the mind — in the unconscious —because there is no mind.
  • More generally: overconfidence in cool new ideas, and exaggeration of what they can do.
  • Over-centralizing around an eliminativist hypothesis or research program in a way that pushes out brainstorming, hypothesis-generation, etc. that isn't easy to fit into that frame. I quote Hempel (1935) here:
[Behaviorism's] principal methodological postulate is that a scientific psychology should limit itself to the study of the bodily behavior with which man and the animals respond to changes in their physical environment, and should proscribe as nonscientific any descriptive or explanatory step which makes use of terms from introspective or 'understanding' psychology, such as 'feeling', 'lived experience', 'idea', 'will', 'intention', 'goal', 'disposition', 'represension'. We find in behaviorism, consequently, an attempt to construct a scientific psychology[.]
  • Simply put: getting the wrong answer. Some errors are more excusable than others, but even if my narrative about why they got it wrong is itself wrong, it would still be important to emphasize that they got it wrong, and could have done much better.
  • The general idea that introspection is never admissible as evidence. It's fine if you want to verbally categorize introspective evidence as 'unscientific' in order to distinguish it from other kinds of evidence, and there are some reasonable grounds for skepticism about how strong many kinds of introspective evidence are. But evidence is still evidence; a Bayesian shouldn't discard evidence just because it's hard to share with other agents.
  • The rejection of folk-psychology language, introspective evidence, or anything else for science-as-attire [LW · GW] reasons.

Idealism emphasized some useful truths (like 'our perceptions and thoughts are all shaped by our mind's contingent architecture') but ended up in a 'wow it feels great to make minds more and more important' death spiral [? · GW].

Behaviorism too emphasized some useful truths (like 'folk psychology presupposes a bunch of falsifiable things about minds that haven't all been demonstrated very well', 'it's possible for introspection to radically mislead us in lots of ways', and 'it might benefit psychology to import and emphasize methods from other scientific fields that have a better track record') but seems to me to have fallen into a 'wow it feels great to more and more fully feel like I'm playing the role of a True Scientist and being properly skeptical and cynical and unromantic about humans' trap.

Replies from: SaidAchmiz
comment by Said Achmiz (SaidAchmiz) · 2019-10-19T19:32:31.775Z · LW(p) · GW(p)

The general idea that introspection is never admissible as evidence. It’s fine if you want to verbally categorize introspective evidence as ‘unscientific’ in order to distinguish it from other kinds of evidence, and there are some reasonable grounds for skepticism about how strong many kinds of introspective evidence are. But evidence is still evidence; a Bayesian shouldn’t discard evidence just because it’s hard to share with other agents.

I find that Dennett’s heterophenomenology squares this circle, fully as much as it can be squared in the absence of actual telepathy (or comparable tech).

Replies from: RobbBB
comment by Rob Bensinger (RobbBB) · 2019-10-19T20:23:36.313Z · LW(p) · GW(p)

"Heterophenomenology" might be fine as a meme for encouraging certain kinds of interesting research projects, but there are several things I dislike about how Dennett uses the idea.

Mainly, it's leaning on the social standards of scientific practice, and on a definition of what "real science" or "good science" is, to argue against propositions like "any given scientist studying consciousness should take into account their own introspective data -- e.g., the apparent character of their own visual field -- in addition to verbal descriptions, as an additional fact to explain." This is meant to serve as a cudgel and bulwark against philosophers like David Chalmers, who claim that introspection reveals further facts (/data/explananda) not strictly translatable into verbal reports.

This is framing the issue as one of social-acceptability-to-the-norms-of-scientists or conformity-with-a-definition-of-"science", whereas correct versions of the argument are Bayesian. (And it's logically rude to not make the Bayesianness super explicit and clear, given the opportunity; it obscures your premises while making your argument feel more authoritative via its association with "science" [LW · GW].)

We can imagine a weird alien race (or alien AI) that has extremely flawed sensory faculties, and very good introspection. A race like that might be able to bootstrap to good science, via leveraging their introspection to spot systematic ways in which their sensory faculties fail, and sift out the few bits of reliable information about their environments.

Humans are plausibly the opposite: as an accident of evolution, we have much more reliable sensory faculties than introspective faculties. This is a generalization from the history of science and philosophy, and from the psychology literature. Moreover, humans have a track record of being bad at distinguishing cases where their introspection is reliable from cases where it's unreliable; so it's hard to be confident of any lines we could draw between the "good introspection" and the "bad introspection". All of this is good reason to require extra standards of evidence before humanity "takes introspection at face value" and admits it into its canon of Established Knowledge.

Personally, I think consciousness is (in a certain not-clarified-here sense) an illusion, and I'm happy to express confidence that Chalmers' view is wrong. But I think Dennett has been uniquely bad at articulating the reasons Chalmers is probably wrong, often defaulting to dismissing them or trying to emphasize their social illegitimacy (as "unscientific").

The "heterophenomenology" meme strikes me as part of that project, whereas a more honest approach would say "yeah, in principle introspective arguments are totally admissible, they just have to do a bit more work than usual because we're giving them a lower prior (for reasons X, Y, Z)" and "here are specific reasons A, B, C that Chalmers' arguments don't meet the evidential bar that's required for us to take the 'autophenomenological' data at face value in this particular case".

Replies from: SaidAchmiz, SaidAchmiz
comment by Said Achmiz (SaidAchmiz) · 2019-10-19T21:51:19.660Z · LW(p) · GW(p)

We can imagine a weird alien race (or alien AI) that has extremely flawed sensory faculties, and very good introspection. A race like that might be able to bootstrap to good science, via leveraging their introspection to spot systematic ways in which their sensory faculties fail, and sift out the few bits of reliable information about their environments.

I don’t think I can imagine this, actually. It seems to me to be somewhat incoherent. How exactly would this race “spot systematic ways in which their sensory faculties fail”? After all, introspection does no good when it comes to correcting errors of perception of the external world…

Or am I misunderstanding your point…?

Replies from: RobbBB
comment by Rob Bensinger (RobbBB) · 2019-10-19T22:13:07.441Z · LW(p) · GW(p)

A simple toy example would be: "You have perfect introspective access to everything about how your brain works, including how your sensory organs work. This allows you to deduce that your external sensory organs provide noise data most of the time, but provide accurate data about the environment anytime you wear blue sunglasses at night."

Replies from: SaidAchmiz
comment by Said Achmiz (SaidAchmiz) · 2019-10-19T22:31:52.262Z · LW(p) · GW(p)

I confess I have trouble imagining this, but it doesn’t seem contradictory, so, fair enough, I take your point.

comment by Said Achmiz (SaidAchmiz) · 2019-10-19T21:49:39.517Z · LW(p) · GW(p)

I don’t read Dennett as referring to social acceptability or “norms of science” (except insofar as those norms are taken to constitute epistemic best practices from a personal standpoint, which I think Dennett does assume to some degree—but no more than is, in my view, warranted).

a more honest approach would say “yeah, in principle introspective arguments are totally admissible, they just have to do a bit more work than usual because we’re giving them a lower prior (for reasons X, Y, Z)”

Sure. Heterophenomenology is that “more work”. Introspective arguments are admissible; they’re admissible as heterophenomenological evidence.

It is indisputably the case that Chalmers, for instance, makes arguments along the lines of “there are further facts revealed by introspection that can’t be translated into words”. But it is not only not indisputably the case, but indeed can’t ever (without telepathy etc., or maybe not even then) be shown to another person, or perceived by another person, to be the case, that there are further facts revealed by introspection that can’t be translated into words.

Indeed it’s not even clear how you’d demonstrate to yourself that what your introspection reveals is real. Certainly you’re welcome to “take introspection’s word for it”—but then you don’t need science of any kind. That I experience what I experience, seems to me to need no demonstration or proof; how can it be false, after all? Even in principle? But then what use is arguing whether a Bayesian approach to demonstrating this not-in-need-of-demonstration fact is best, or some other approach? Clearly, whatever heterophenomenology (or any other method of investigation) might be concerned with, it’s not that.

But now I’m just reiterating Dennett’s arguments. I guess what I’m saying is, I think your responses to Dennett are mostly mis-aimed. I think the rebuttals are already contained in what he’s written on the subject.

Replies from: RobbBB
comment by Rob Bensinger (RobbBB) · 2019-10-19T23:22:04.852Z · LW(p) · GW(p)
It is indisputably the case that Chalmers, for instance, makes arguments along the lines of “there are further facts revealed by introspection that can’t be translated into words”. But it is not only not indisputably the case

What does "indisputably" mean here in Bayesian terms? A Bayesian's epistemology is grounded in what evidence that individual has access to, not in what disputes they can win. When Chalmers claims to have "direct" epistemic access to certain facts, the proper response is to provide the arguments for doubting that claim, not to play a verbal sleight-of-hand like Dennett's (1991, emphasis added):

You are not authoritative about what is happening in you, but only about what seems to be happening in you, and we are giving you total, dictatorial authority over the account of how it seems to you, about what it is like to be you. And if you complain that some parts of how it seems to you are ineffable, we heterophenomenologists will grant that too. What better grounds could we have for believing that you are unable to describe something than that (1) you don’t describe it, and (2) confess that you cannot? Of course you might be lying, but we’ll give you the benefit of the doubt.

It's intellectually dishonest of Dennett to use the word "ineffable" here to slide between the propositions "I'm unable to describe my experience" and "my experience isn't translatable in principle", as it is to slide between Nagel's term of art "what it's like to be you" and "how it seems to you".

Again, I agree with Dennett that Chalmers is factually wrong about his experience (and therefore lacks a certain degree of epistemic "authority" with me, though that's such a terrible way of phrasing it!). There are good Bayesian arguments against trusting autophenomenology enough for Chalmers' view to win the day (though Dennett isn't describing any of them here), and it obviously is possible to take philosophers' verbal propositions as data to study (cf. also the meta-problem of consciousness), but it's logically rude to conceal your cruxes, pretend that your method is perfectly neutral and ecumenical, and let the "scientificness" of your proposed methodology do the rhetorical pushing and pulling.

but indeed can’t ever (without telepathy etc., or maybe not even then) be shown to another person, or perceived by another person, to be the case, that there are further facts revealed by introspection that can’t be translated into words.

There's a version of this claim I agree with (since I'm a physicalist), but the version here is too strong. First, I want to note again that this is equating group epistemology with individual epistemology. But even from a group's perspective, it's perfectly possible for "facts revealed by introspection that can't be translated into words" to be transmitted between people; just provide someone with the verbal prompts (or other environmental stimuli) that will cause them to experience and notice the same introspective data in their own brains.

If that's too vague, consider this scenario as an analogy: Our universe is a (computable) simulation, running in a larger universe that's uncomputable. Humans are "dualistic" in the sense that they're Cartesian agents outside the simulation whose brains contain uncomputable subprocesses, but their sensory experiences and communication with other agents is all via the computable simulation. We could then imagine scenarios where the agents have introspective access to evidence that they're performing computations too powerful to run in the laws of physics (as they know them), but don't have output channels expressive enough to demonstrate this fact to others in-simulation; instead, they prompt the other agents to perform the relevant introspective feat themselves.

The other agents can then infer that their minds are plausibly all running on physics that's stronger than the simulated world's physics, even though they haven't found a directly demonstrate this (e.g., via neurosurgery on the in-simulation pseudo-brain).

Indeed it’s not even clear how you’d demonstrate to yourself that what your introspection reveals is real.

You can update upward or downward about the reliability of your introspection (either in general, or in particular respects), in the same way you can update upward or downward about the reliability of your sensory perception. E.g., different introspective experiences or faculties can contradict each other, suggest their own unreliability ("I'm introspecting that this all feels like bullshit..."), or contradict other evidence sources.

Replies from: SaidAchmiz, SaidAchmiz, RobbBB
comment by Said Achmiz (SaidAchmiz) · 2019-10-20T06:43:19.007Z · LW(p) · GW(p)

What does “indisputably” mean here in Bayesian terms? A Bayesian’s epistemology is grounded in what evidence that individual has access to, not in what disputes they can win.

Ok… before I respond with anything else, I want to note that this is hardly a reasonable response. “Indisputably” is a word that has several related usages, and while indeed one of them is something sort of like “you won’t actually win any actual debates if you try to take the opposite position”, do you really think the most plausible way to interpret what I said is to assume that that is the usage I had in mind? Especially after I wrote:

I don’t read Dennett as referring to social acceptability or “norms of science” (except insofar as those norms are taken to constitute epistemic best practices from a personal standpoint, which I think Dennett does assume to some degree—but no more than is, in my view, warranted).

So it should be clear that I’m not talking about winning debates, or social acceptability, or any such peripheral nonsense. I am, and have been throughout this discussion, talking about epistemology. Do I really need to scrupulously eschew such (in theory ambiguous but in practice straightforward) turns of phrase like “indisputably”, lest I be treated to a lecture on Bayesian epistemology?

If you really don’t like “indisputably”, substitute any of the following, according to preference:

  • plainly
  • manifestly
  • obviously
  • clearly
  • certainly
  • indubitably
  • incontrovertibly
  • with nigh-perfect certainty

… etc., etc.

comment by Said Achmiz (SaidAchmiz) · 2019-10-21T12:15:14.027Z · LW(p) · GW(p)

And now, a substantive response:

When Chalmers claims to have “direct” epistemic access to certain facts, the proper response is to provide the arguments for doubting that claim, not to play a verbal sleight-of-hand like Dennett’s (1991, emphasis added):

You are not authoritative about what is happening in you, but only about what seems to be happening in you, and we are giving you total, dictatorial authority over the account of how it seems to you, about what it is like to be you. And if you complain that some parts of how it seems to you are ineffable, we heterophenomenologists will grant that too. What better grounds could we have for believing that you are unable to describe something than that (1) you don’t describe it, and (2) confess that you cannot? Of course you might be lying, but we’ll give you the benefit of the doubt.

It’s intellectually dishonest of Dennett to use the word “ineffable” here to slide between the propositions “I’m unable to describe my experience” and “my experience isn’t translatable in principle”, as it is to slide between Nagel’s term of art “what it’s like to be you” and “how it seems to you”.

First of all, how in the world could you possibly know that your experience isn’t translatable in principle? That you can’t describe it—that you of course can know. But what additional meaning can it even have, to say that you can’t describe it, and on top of that, it “isn’t translatable in principle”? What does that even mean?

As far as I can tell, Dennett isn’t sliding between anything. There’s just the one meaning: you can’t describe some experience you’re having.

Secondly, it’s not clear that this paragraph is a response to claims about having “‘direct’ epistemic access to certain facts”. (I’d have to reread Consciousness Explained to see the context, but as quoted it seems a bit of a non sequitur.)

… it’s logically rude to conceal your cruxes, pretend that your method is perfectly neutral and ecumenical, and let the “scientificness” of your proposed methodology do the rhetorical pushing and pulling.

I confess I don’t really have much idea what you’re saying here. What’s Dennett concealing, exactly…?

but indeed can’t ever (without telepathy etc., or maybe not even then) be shown to another person, or perceived by another person, to be the case, that there are further facts revealed by introspection that can’t be translated into words.

There’s a version of this claim I agree with (since I’m a physicalist), but the version here is too strong. First, I want to note again that this is equating group epistemology with individual epistemology.

I wasn’t talking about group epistemology here at all, much less equating it with anything.

But even from a group’s perspective, it’s perfectly possible for “facts revealed by introspection that can’t be translated into words” to be transmitted between people; just provide someone with the verbal prompts (or other environmental stimuli) that will cause them to experience and notice the same introspective data in their own brains.

This clearly won’t do; how will you ever know that the verbal prompts (or etc.) are causing the other person to experience, much less to notice, the same “introspective data” in their brain as you experienced and noticed in yours? (How exactly do you even guarantee comparability? What does “same” even mean, across individuals? People vary, you know; and it seems fairly likely even from what we know now, that capacity to experience certain things is present to widely varying degrees in people…)

Why, there are entire reams of philosophy dedicated to precisely this very thorny challenge! (Google “spectrum inversion” sometime…) And in fact I once saw this principle play out in my own life. A musically inclined friend of mine was attempting to teach me the basics of music theory. When his initial explanations got nowhere, we opened someone’s laptop and loaded up a website where you could click buttons and play certain chords or combinations of tones. My friend clicked some buttons, played some chords, and asked me to describe what I heard, which I did… only to see my friend react with astonishment, because what I heard and what he heard turned out to be quite different. (As we later discovered, I have some interesting deficiencies/abnormalities in auditory processing, having to do, inter alia, with ability to perceive pitch.)

Now, how do you propose to cause me to experience “the same introspective data” that my friend experiences when he hears the tones and chords in question—or vice versa? What stimuli, exactly, shall you use—and how would you discover what they might be? What function, precisely, reliably maps arbitrary (stimulus X, individual A) pairs to (stimulus Y, individual B) pairs, such that the “introspective data” that is experienced (and noticed) as a result is the “same” in both cases of a set? And having on hand a candidate such function, how exactly would you ever verify that it is really the desired thing?

If that’s too vague, consider this scenario as an analogy: …

I find such fanciful analogies almost uniformly uninformative, and this one, I’m afraid, is no exception. Even if I were to stretch my brain to imagine this sort of scenario (which is not easy), and carefully consider its implications (which is quite challenging), and take the further step of drawing a conclusion about whether the given hypothetical would indeed work as you say (in which I would have quite low confidence), nevertheless it would still be entirely unclear whether, and how, the analogy mapped back to our actual world, and whether any of the reasoning and the conclusion still held. Best to avoid such things.

Indeed it’s not even clear how you’d demonstrate to yourself that what your introspection reveals is real.

You can update upward or downward about the reliability of your introspection (either in general, or in particular respects), in the same way you can update upward or downward about the reliability of your sensory perception. E.g., different introspective experiences or faculties can contradict each other, suggest their own unreliability (“I’m introspecting that this all feels like bullshit...”), or contradict other evidence sources.

What if there is no “contradiction”, as such? Surely it’s possible for introspection to be deficient or entirely misleading even so? In any case, if introspection is corrigible by comparison with “other evidence sources” (by which you presumably mean, sense data, and experimental and various other observational information acquired via sense data, etc.), then you can hardly be said to have “‘direct’ epistemic access” to anything via said introspection…

comment by Rob Bensinger (RobbBB) · 2019-10-20T02:57:39.561Z · LW(p) · GW(p)
When Chalmers claims to have "direct" epistemic access to certain facts, the proper response is to provide the arguments for doubting that claim, not to play a verbal sleight-of-hand like Dennett's (1991, emphasis added):

Chalmers' The Conscious Mind was written in 1996, so this is wrong. The wrongness doesn't seem important to me. (Jackson and Nagel were 1979/1982, and Dennett re-endorsed this passage in 2003.)

comment by Chris_Leong · 2019-10-12T13:22:58.795Z · LW(p) · GW(p)

I appreciated this post for explaining Berkeley's beliefs really clearly to me. I never knew what he was going on about before.

comment by TAG · 2019-10-26T13:54:46.166Z · LW(p) · GW(p)

I'd like to put forward a lukewarm defence of Kant as a friend of science.

Kant is on board with the idea that common sense observations like "refrigerators exist", or a car has four wheels" are true... enough.. In context. He calls it "emprical realism". He summarises his system as "empirical realism and transcendental idealism". Not as "you can't know anything".

Empirical realism means perceptions are based on information entering the mind via the sense organs. Kant is in agreement with basic scientific realism on this point. He also emphasised the realist side of his philosophy by adding a "Refutation of Idealism" to the second version of the Critique of pure Reason.

Transendental idealism means all specific human perceptions are moulded by the general form of human perception and there is no way to backtrack to a raw form. We can concede that humans and bats perceive the world differently.. although Kant goes further than most.

The kind of knowledge he says you can't have is knowledge of the thing in itself, which in modern terms would mean something like knowledge that is not relative to some conceptual framework or way of perceiving. Physicalism doesn't refute that in the least, because it is explicitly based on using physical science as its framework.

Why does Kant include Transcendental Idealism and not go for 100% realism? Because he is trying to rescue science from some philosophical criticisms! Berkely and other idealists attacked the occult, invisible nature of Newtonian gravity and absolute space. It seemed an embarassment for empirical science to be based on invisible things. Additionally, Hume attacked the cause-effect relation as indiscernable from mere temporal succession

Transcendental Idealism is a stepping stone to the central point of Kants system, the argument that space and time and causality and substance are categories that the human mind uses to organise its sense data, not external realities. He described this radical view with a term borrowed from science: a Copernican Revolution.

Kant was impressed by Newton's physics (and lectured on astronomy himself). He believed that nothing less than a necessary connection between cause and effect would do justice to Newton's clockwork determinism. His contemporaries were happy to accept that logical implications were necessary, but Hume had argued that fire did not imply smoke.

To meet this difficulty, Kant restructures the traditional logical/empirical distinction into a fourfold grid of analytical/synthetic times apriori/a posteriori, and places causality in the "synthetic apriori" quadrant. By any account, something is necessary if it true in all possible cases. Kant tries to restore the necessity of causality by arguing that cause and affect (and the other categories of transcendental idealism) must hold on all possible cases of human perception.

So the idealistic aspects of Kants are there to support the science of his day against sceptical attacks, and are accompanied by realistic elements.

Whether his strange and ingenious scheme is still necessary is very open to debate. Neither physical determinism nor logical necessity are as important as they were; our notion of empiricism includes a strong element of abduction, or influence to the best explanation; we end to have multiple theories rather than different paradigms; and we have become comfortable with invisible entities.

Nonetheless, some of Kant's ideas remain open question, for instance the ontological status of time.

Replies from: TAG, RobbBB
comment by TAG · 2023-05-18T14:29:33.626Z · LW(p) · GW(p)

Not only is Kant basing his philosophical views on the science of his day, but Kantian philosophical conclusions are being promoted as science, by scientists.

"The idea that our senses (or, indeed, those of any conscious agent) don’t disclose reality is the main idea of The Case Against Reality by Donald Hoffman, a cognitive scientist at UC Irvine, who argues that in addition to its ancient pedigree, this counterintuitive view of reality and perception is also supported by the latest science."

https://docs.google.com/document/d/1AtGIIv371v0Yu35eNsIxJr67dw4SHOiGdKrqmoKt2hg/edit

comment by Rob Bensinger (RobbBB) · 2019-10-26T16:51:08.452Z · LW(p) · GW(p)

I agree that Kant thought of himself as trying to save science from skepticism (e.g., Hume) and weird metaphysics (e.g., Berkeley), and I'm happy you're trying to make it easier to pass Kant's Ideological Turing Test.

Transendental idealism means all specific human perceptions are moulded by the general form of human perception and there is no way to backtrack to a raw form. [...]
The kind of knowledge he says you can't have is knowledge of the thing in itself, which in modern terms would mean something like knowledge that is not relative to some conceptual framework or way of perceiving. Physicalism doesn't refute that in the least, because it is explicitly based on using physical science as its framework.

I have two objections:

(1) Physicalism does contradict the claim "there is no way to backtrack to a raw form", if this is taken to mean we should be agnostic about whether things are (really, truly, mind-independently) physical.

I assert that the "raw form" of an electron, insofar as physics is accurate, is just straightforwardly and correctly described by physics; and unless there's a more fundamental physical account of electrons we have yet to discover, physics is plausibly (though I doubt we can ever prove this) a complete description of electrons. There may not be extra features that we're missing.

(2) Modern anti-realist strains, similar to 19th-century idealism, tend to slide between these three claims:

  • "We can't know things about ultimate reality without relying on initially unjustified knowledge/priors/cognitive machinery."
  • "We can't know things about ultimate reality."
  • "(We can know that) ultimate reality is wildly different from reality-as-we-conceive-of-it."

The first claim is true, but the second and third claims are false.

This sliding is probably the real thing we have Kant to thank for, and the thing that's made anti-realist strains so slippery and hard to root out; Berkeley was lucid enough to unequivocally avoid the above leaps.

Quoting Allen Wood (pp. 63-64, 66-67):

The doctrine can even be stated with apparent simplicity: We can have cognition of appearances but not of things in themselves. But it is far from clear what this doctrine means, and especially unclear what sort of restriction it is supposed to place on our knowledge.
Some readers of Kant have seen the restriction as trivial, so trivial as to be utterly meaningless, even bordering on incoherence. They have criticized Kant not for denying that we can know 'things in themselves' but rather for thinking that the notion of a 'thing in itself' even makes sense. If by a 'thing in itself' we mean a thing standing outside any relation to our cognitive powers, then of course it seems impossible for us to know such things; perhaps it is even self-contradictory to suppose that we could so much as think of them.
Other readers have seen transcendental idealism as a radical departure from common sense, a form of skepticism at least as extreme as any Kant might have been trying to combat. To them it seems that Kant is trying (like Berkeley) to reduce all objects of our knowledge to mere ghostly representations in our minds. He is denying us the capacity to know anything whatever about any genuine (that is, any extra-mental) reality. [...]
I think much of the puzzlement about transcendental idealism arises from the fact that Kant himself formulates transcendental idealism in a variety of ways, and it is not at all clear how, or whether, his statements of it can all be reconciled, or taken as statements of a single, self-consistent doctrine. I think Kant's central formulations suggest two quite distinct and mutually incompatible doctrines. [...]
Some interpreters of Kant, when they become aware of these divergences, respond by saying that there is no significant difference between the two interpretations, that they are only 'two ways of saying the same thing.' These interpreters are probably faithful to Kant's intentions, since it looks as if he thought the two ways of talking about appearances and things in themselves are interchangeable and involve no difference in doctrine. But someone can intend to speak self-consistently and yet fail to do so; and it looks like this is what has happened to Kant in this case.

In particular, here's Wood on why Kant is sometimes saying 'we can't know about the world outside our heads', not just 'we can't have knowledge without relying on some conceptual framework or way of perceiving' (p. 64):

Kant often distinguishes appearances from things in themselves through locutions like the following: "What the objects may be in themselves would still never be known through the most enlightened cognition of their appearance, which alone is given to us" (KrV A43/B60). "Objects in themselves are not known to us at all, and what we call external objects are nothing other than mere representations of our sensibility, whose form is space, but whose true correlate, i.e. the thing in itself, is not and cannot be cognized through them" (KrV A30/B45).
Passages like these suggest that things existing in themselves are entities distinct from 'their appearances' -- which are subjective states caused in us by these things. Real things (things in themselves) cause appearances. Appearances have no existence in themselves, being only representations in us. "Appearances do not exist in themselves, but only relative to the [subject] insofar as it has senses" (KrV B164). "But we should consider that bodies are not objects in themselves that are present to us, but rather a mere appearance of who knows what unknown object; that motion is not the effect of this unknown cause, but merely the appearance of its influence on our senses; that consequently neither of these is something outside us, but both are merely representations in us" (KrV A387).

Whereas (p. 65):

In other passages, transcendental idealism is formulated so as to present us with a very different picture. [...] Here Kant does not distinguish between two separate entities, but rather between the same entity as it appears (considered in relation to our cognitive faculties) and as it exists in itself (considered apart from that relation). [...]
On the identity interpretation, appearances are not merely subjective entities or states in our minds; they do have an existence in themselves. The force of transcendental idealism is not to demote them, so to speak, from reality to ideality, but rather to limit our cognition of real entities to those features of them that stand in determinate relations to our cognitive faculties.
Replies from: TAG, TAG, TAG
comment by TAG · 2019-10-27T20:02:21.590Z · LW(p) · GW(p)

In particular, here’s Wood on why Kant is sometimes saying ‘we can’t know about the world outside our heads’, not just ‘we can’t have knowledge without relying on some conceptual framework or way of perceiving’ (p. 64):

Kant often distinguishes appearances from things in themselves through locutions like the following: “What the objects may be in themselves would still never be known through the most enlightened cognition of their appearance, which alone is given to us” (KrV A43/B60). “Objects in themselves are not known to us at all, and what we call external objects are nothing other than mere representations of our sensibility, whose form is space, but whose true correlate, i.e. the thing in itself, is not and cannot be cognized through them” (KrV A30/B45).

I dont think the quoted passage supports your claim at all. If the object "in itself" is the object outside of any conceptual or perceptual framework, then K. Is just that all knowledge occurs within a framework, and knowledge outside a framework is impossible.

I could, however, see how Kant could be read as denying any knowledge of external objects, under circumstances where no particular meaning is assigned to "in itself".

Replies from: RobbBB
comment by Rob Bensinger (RobbBB) · 2019-10-28T01:05:57.674Z · LW(p) · GW(p)

I mean "But we should consider that bodies are [...] a mere appearance of who knows what unknown object; that motion is not the effect of this unknown cause, but merely the appearance of its influence on our senses; that consequently neither of these is something outside us, but both are merely representations in us" seems pretty unambiguous to me. Kant isn't saying here that 'we can only know stuff about mind-independent objects by using language and concepts and frameworks' in this passage; he's saying 'we can only know stuff about mere representations inside of us'.

Kant passages oscillate between making sense under one of these interpretations or the other (or neither):

  • the "causality interpretation", which says that things-in-themselves are objects that cause appearances, like a mind-independent object causes an experience in someone's head. If noumena are the "true correlates" of phenomena, while phenomena are nothing but subjective experiences, then this implies that we really don't know anything about the world outside our heads. You can try to squirm out of this interpretation by asserting that words like "empirical" and "world" should be redefined to refer to subjective experiences in our heads, but this is just playing with definitions.
  • the "identity interpretation", which says that things-in-themselves are the same objects as phenomena, just construed differently.

Quoting Wood (66-67, 69-70):

Yet the two interpretations appear to yield very different (incompatible) answers to the following three questions:
1. Is an appearance the very same entity as a thing in itself? The causality interpretation says no, the identity interpretation says yes.
2. Are appearances caused by things in themselves? The causality interpretation says yes, the identity interpretation says no.
3. Do the bodies we cognize have an existence in themselves? The causality interpretation says no, the identity interpretation says yes.
[... N]o entity stands to itself in the relation of cause to effect. Transcendental idealism is no intelligible doctrine at all if it cannot give self-consistent answers to the above three questions. [...]
Kant occasionally tries to combine "causality interpretation" talk with "identity interpretation" talk. When he does, the result is simply nonsense and self-contradiction:
"I say that things as objects of our senses existing outside us are given, but we know nothing of what they may be in themselves, cognizing only their appearances, that is, the representations which they cause in us by affecting our senses. Consequently, I grant by all means that there are bodies outside us, that is, things which, though quite unknown to us as to what they are in themselves, we still cognize by the representations which their influence on our sensibility procures us, and which we call bodies, a term signifying merely the appearance of the thing which is unknown to us but not the less actual. (P 4:289)
The first sentence here says that objects of the senses are given to our cognition, but then denies that we cognize these objects, saying instead that we cognize an entirely different set of objects (different from the ones he has just said are given). The second sentence infers from this that there are bodies outside us, but proceeds to say that it is not these bodies (that is, the entities Kant has just introduced to us as 'bodies') that we call 'bodies', but rather bodies are a wholly different set of entities. Such Orwellian doubletalk seems to be the inevitable result of trying to combine the causality interpretation with the identity interpretation while supposing that they are just two ways of saying the same thing. [...]
Kant of course denies that we can ever have cognition of an object as it is in itself, because we can have no sensible intuition of it -- as it is in itself. But he seems to regard it as entirely permissible and even inevitable that we should be able to think the phenomenal objects around us solely through pure concepts of the understanding, hence as they are in themselves. If I arrive at the concept of a chair in the corner first by cognizing it empirically and then by abstracting from those conditions of cognition, so that I think of it existing in itself outside those conditions, then it is obvious that I am thinking of the same object, not of two different objects. It is also clear that when I think of it the second way, I am thinking of it, and not of its cause (if it has one). From this point of view, the causality interpretation seems utterly unmotivated and even nonsensical.
The problem arises, however, because Kant also wants to arrive at the concept of a thing existing in itself in another way. He starts from the fact that our empirical cognition results from the affection of our sensibility by something outside us. This leads him to think that there must be a cause acting on our sensibility from outside, making it possible for us to intuit appearances, which are then conceived as the effects of this cause.
Of course it would be open to him to think of this for each case of sensible intuition as the appearance acting on our sensibility those a wholly empirical causality. But Kant apparently arrived at transcendental idealism in part by thinking of it as a revised version of the metaphysics of physical influence between substances that he derived from Crusius. Thus sensible intuition is sometimes thought of as the affection of our senses by an object not as an appearance but as a thing in itself, and transcendental idealism is thought of as having to claim (inconsistently) that we are to regard ourselves (as things in themselves) as being metaphysically influenced by things in themselves.
Such a metaphysics would of course be illegitimately transcendent by the standards of the Critique, but Kant unfortunately appears sometimes to think that transcendental idealism is committed to it, and many of his followers down to the present day seem addicted to the doctrine that appears to be stated in the letter of those texts that express that thought, despite the patent nonsense they involve from the critical point of view. The thing in itself is then taken to be this transcendent cause affecting our sensibility as a whole, and the appearance is seen as the ensemble of representations resulting from its activity on us.
comment by TAG · 2019-10-27T18:03:14.467Z · LW(p) · GW(p)

We can’t know things about ultimate reality without relying on initially unjustified knowledge/priors/cognitive machinery.”

I think that is actually two related problems.

1a. A general problem of founding knowledge.. the Epistemic Circle or Munchausen Trilemma...which applies to everything, even maths.

1b. The specifically Kantian problem, which is limited to knowledge of the external world.

This sliding is probably the real thing we have Kant to thank for, and the thing that’s made anti-realist strains so slippery and hard to root out;

I think it is pretty unfair to blame the whole problem on Kant. He is a much less clear writer than Berkely, but then he is expressing much more complex ideas. Also, a lot of the sliding is surely cause by non-philosophers summarising complex ideas into one line slogans.

comment by TAG · 2019-10-27T17:25:08.093Z · LW(p) · GW(p)

Physicalism does contradict the claim “there is no way to backtrack to a raw form”, if this is taken to mean we should be agnostic about whether things are (really, truly, mind-independently) physical

"Mind independent" and "physical" don't mean quite the same thing, and there are multiple ways of cashing out "physical".

I assert that the “raw form” of an electron, insofar as physics is accurate, is just straightforwardly and correctly described by physics;

What "raw" means is "absent any conceptual framework". Making that substitution, your statement is a clear contradiction,along the lines of "We have a way of knowing what an electron is outside of any conceptual framework, and that way is to use the conceptual framework known as physics".

But even given the correctness of the tautologous version of Kant, a version of physicalism can still be asserted as, for instance, "ideal physical science is the correct framework".

If by a ‘thing in itself’ we mean a thing standing outside any relation to our cognitive powers, then of course it seems impossible for us to know such things; perhaps it is even self-contradictory to suppose that we could so much as think of them.

Well, indeed. The question is: what follows from this truth being merely tautologous? One thing that follows is the falsehood of strong physicalism. (I believe that a solution to the hard problem of consciousness follows that in turn, but that is a topic for another day).

Replies from: RobbBB
comment by Rob Bensinger (RobbBB) · 2019-10-27T23:08:08.635Z · LW(p) · GW(p)

I claim that the most natural interpretation of "[Transcendental] idealism means all specific human perceptions are moulded by the general form of human perception and there is no way to backtrack to a raw form." is that there's no way to backtrack from our beliefs, impressions, and perceptions to ultimate reality. That is, I'm interpreting "backtrack" causally: the world causes our perceptions, and backtracking would mean reconstructing what the ultimate, outside-our-heads, existed-before-humanity reality is like before we perceive or categorize it. (Or perhaps backtracking causally to the initial, relatively unprocessed sense-data our brains receive.)

In those terms, we know a ton about ultimate, outside-our-heads reality (and a decent amount about how the brain processes new sensory inputs), and there's no special obstacle to backtracking from our processed sense data to the raw, unprocessed real world. (Our reasoning faculties do need to be working OK, but that's true for our ability to learn truths about math, about our own experiences, etc. as well. Good conclusions require a good concluder.)

If instead the intended interpretation of "backtrack to a raw form" is "describe something without describing it", "think about something without thinking about it", or "reason about something without reasoning about it", then your original phrasing stops making sense to me.

Take the example of someone standing by a barn. They can see the front side of the barn, but they've never observed the back side. At noon, you ask them to describe their subjective experience of the barn, and they do so. Then you ask them to "backtrack to the raw form" beyond their experience. They proceed to start describing the full quantum state of the front of the barn as it was at noon (taking into account many-worlds: the currently-speaking observer has branched off from the original observer).

Then you go, "No, no, I meant describe something about the barn as it exists outside of your conceptual schemes." And the person repeats their quantum description, which is a true description regardless of the conceptual scheme used; the quantum state is in the world, not in my brain or in my concepts.

Then you go, "No, I meant describe an aspect of the barn that transcends your experiences entirely; not a property of the barn that caused your experience, but a property unconnected to your experience." And the person proceeds to conjecture that the barn has a back side, even though they haven't seen it; and they start speculating about likely properties the back side may have.

Then you go, "No! I meant describe something about the barn without using your concepts in the description." Or: "Describe something that bears no causal relation to your cognition whatsoever, like a causally inert quiddity that in no way interacts with any of the kinds of things you've ever experienced or computed."

And the person might reply: Well, I can say that such a thing would be a causally inert quiddity, as you say; and then perhaps I can't say much more than that, other than to drill down on what the relevant terms mean. Or, if the requirement is to describe a thing without describing it, then obviously I can't do that; but that seems like an even more trivial observation.

Why would the request to "describe something without describing it" ever be phrased as "backtracking to a raw form"? There's no "backtracking" involved, and we aren't returning to an earlier "raw" or unprocessed thing, since we're evidently not talking about an earlier (preconceptual) cognition that was subsequently processed into a proper experience; and since we're evidently not talking about the physical objects outside our heads that are the cause and referent for our thoughts about them.

I claim that there's an important equivocation at work in the idealist tradition between "backtracking" or finding a more "raw" or ultimate version of a thing, and "describe a thing without describing it". I claim that these only sound similar because of the mistake in Berkeley's master argument: confusing the ideas "an electron (i.e., an object) that exists outside of any conceptual framework" and "an 'electron' (i.e., a term or concept) that exists outside of any conceptual framework". I claim that the very temptation to use 'Ineffable-Thingie'-reifying phrasings like "there is no way to backtrack to a raw form" and "what an electron is outside of any conceptual framework", is related to this mistake.

Phrasing it as "We can't conceive of an electron without conceiving of it" makes it sound trivial, whereas the way of speaking that phrases things almost as though there were some object in the world (Kant's 'noumena') that transcends our conceptual frameworks and outstrips our every attempt to describe it, makes it sound novel and important and substantive. (And makes it an appealing Inherently Mysterious Thing [LW · GW] to worship.)

Replies from: RobbBB, TAG
comment by Rob Bensinger (RobbBB) · 2019-10-27T23:11:14.391Z · LW(p) · GW(p)

The reason I'm focusing on this is that I think some of the phrasings you chose in trying to summarize Kant (and translate or steelman his views) are sliding between the three different claims I described above:

[1] "We can't know things about ultimate reality without relying on initially unjustified knowledge/priors/cognitive machinery."
[2] "We can't know things about ultimate reality."
[3] "(We can know that) ultimate reality is wildly different from reality-as-we-conceive-of-it."

E.g., you say

The kind of knowledge he says you can't have is knowledge of the thing in itself, which in modern terms would mean something like knowledge that is not relative to some conceptual framework or way of perceiving

In treating all these claims as equivalent, you're taking a claim that sounds at first glance like 2 ("you can't have knowledge of the thing in itself"), and identifying it with claims that sound like either 1 or 3 ("you can't have knowledge that is not relative to some conceptual framework or way of perceiving," "you can't have knowledge of the real world that exists outside our concepts", "space/time/etc. are things our brains make up, not ultimately real things").

I think dissecting these examples helps make it easier to see how a whole continent could get confused about Berkeleian master-argument-syle reasoning for 100-200 years, and get confused about distinctions like 'a thought you aren't thinking' vs. 'an object-of-thought you aren't thinking about'.

Replies from: TAG
comment by TAG · 2019-11-01T11:29:34.616Z · LW(p) · GW(p)

The reason I’m focusing on this is that I think some of the phrasings you chose in trying to summarize Kant (and translate or steelman his views) are sliding between the three different claims I described above:

[1] “We can’t know things about ultimate reality without relying on initially unjustified knowledge/priors/cognitive machinery.” [2] “We can’t know things about ultimate reality.” [3] “(We can know that) ultimate reality is wildly different from reality-as-we-conceive-of-it.”

With regard to Kant's claims, there is no "sliding" because he asserts all 3 of those, and provides separate arguments for them. In particular, [3] is supported by the Antinomies, which purport to show that spatiality is not a property of objective reality. He is not deriving everything from a single argument, any other than Berkeley is.

comment by TAG · 2019-10-29T19:19:07.864Z · LW(p) · GW(p)

I claim that there’s an important equivocation at work in the idealist tradition between “backtracking” or finding a more “raw” or ultimate version of a thing, and “describe a thing without describing it”.

I can't see why equivocation is helpful. If you want to ascertain the existence of some kind of noumena, you need to distinguish the thing you can do -- come up with a theory of the causes of your perceptions as external physical things -- from the thing you can't do -- get outside the map entirely.

We can’t conceive of an electron without conceiving of it” makes it sound trivial, whereas the way of speaking that phrases things almost as though there were some object in the world (Kant’s ‘noumena’) that transcends our conceptual frameworks and outstrips our every attempt to describe it, makes it sound novel and important and substantive.

The significant-sounding claim does indeed follow from the trivial sounding one. That makes it a good argument. Good arguments should draw non-obvious conclusions from well-founded premises.

comment by shminux · 2019-10-13T02:09:18.724Z · LW(p) · GW(p)
I still intermittently run into people who claim that there's no such thing as reality or truth;

This sounds... strawmanny. "Reality and truth are not always the most useful concepts and it pays to think in other ways at times" would be a somewhat more charitable representation of non-realist ideas.


Replies from: RobbBB, Chris_Leong
comment by Rob Bensinger (RobbBB) · 2019-10-13T04:20:49.488Z · LW(p) · GW(p)

I think my characterization is accurate, but maybe guilty of weak-manning: I'm recounting a salient recent (long) conversation with laypeople, rather than attempting a representative survey of non-realists or trying to find the best proponents.

I had in mind a small social gathering I attended (without any conscious effort to seek out and find non-realists) where most of the people in the room voiced disagreement with my claim that truth is a coherent idea, that some entities aren't social or psychological constructs, that some methods for learning things are more objective/reasonable/justified than others, and so on.

I tried to find common ground on the most basic claims I could think of, like "OK, but we can at least agree that something is real, right? There's, like, stuff actually going on?" I wasn't successful. And I think I'm pretty good at not straw-manning people on these issues; I'm used to drawing pretty fine distinctions between pretty out-there ontological and epistemological views. (E.g., I'm perfectly happy to try to tease apart the nuances of thinkers like Parmenides, Nagarjuna, Zhuangzi, Sextus, William James, Dharmakirti, Schopenhauer, Jonathan Schaffer, Graham Priest, Sartre, Berkeley. This stuff is interesting, even if I put no stock in it.)

To my ear, "it pays to think in terms other than reality/truth sometimes" sound too weak on its own to count as 'anti-realism'. If I think it's ever (cognitively?) useful to read fiction, or explore fake frameworks, or just take a nap and clear my head, that already seems to qualify. I'm happy to hear more about what you have in mind, though, regardless of what labels fit best.

Replies from: ChristianKl
comment by ChristianKl · 2019-10-13T20:32:04.360Z · LW(p) · GW(p)
And I think I'm pretty good at not straw-manning people on these issues; I'm used to drawing pretty fine distinctions between pretty out-there ontological and epistemological views.

To have a conversation where you can draw fine distinctions both parties of the discussion need to be willing to have that conversation. You might have approached the issue as a mistake theorist while they were more in the conflict theorist mindset and didn't want to yield any possible ground.

I remember in a book about applied ontology by a author who saw himself as a realist while the statement that "homo sapiens is a species" was seen as a statement about reality while "Alice sake" in the sense of "He did X for Alice's sake" is not something that really exists. ( I think it was in Applied Ontology: An Introduction by Katherine Munn and Barry Smith)

In contrast to that non-realist ontologists consider the idea of a species to be constructed and not one about reality. What's real about the statement "homo sapiens is a species" is not something that's directly made up of atoms and physics but the realist position by applied ontologists like Barry Smith is still that it's real.

comment by Chris_Leong · 2019-10-14T04:17:17.078Z · LW(p) · GW(p)

It's something people say, but don't necessarily fully believe

Replies from: TAG
comment by TAG · 2019-10-17T07:25:45.776Z · LW(p) · GW(p)

Yes, anti realists still need to do the things that words like "good" and "true" do - - praise and condemn, and do on. But they are unwilling to use them, which leads to a euphemism treadmill, where "false" is substituted with "problematic", "evil" with "toxic".

comment by Y-S.Ko · 2019-10-14T18:09:19.430Z · LW(p) · GW(p)

1. 19th century German version:

Origin of knowledge.- Over immense periods of time the intellect produced nothing but errors. A few of these proved to be useful and helped to preserve the species: those who hit upon or inherited these had better luck in their struggle for themselves and their progeny. Such erroneous articles of faith, which were continually inherited. until they became almost part of the basic endowment of the species, include the following: that there are enduring things; that there are equal things; that there are things, substances. bodies; that a thing is what it appears to be; that our will is free; that what is good for me is also good in itself. It was only very late that such propositions were denied and doubted; it was only very late that truth emerged-as the weakest form of knowledge. It seemed that one was unable to live with it: our organism was prepared for the opposite; all its higher functions, sense perception and every kind of sensation worked with those basic errors which had been incorporated since time immemorial. Indeed, even in the realm of knowledge these propositions became the norms according to which "true" and "untrue" were determined—down to the most remote regions of logic. Thus the strength of knowledge does not depend on its degree of truth but on its age, on the degree to which it has been incorporated. on its character as a condition of life.
-- Friedrich Nietzsche

2. 21st century evolutionary theory version:

The classic argument is that those of our ancestors who saw more accurately had a competitive advantage over those who saw less accurately and thus were more likely to pass on their genes that coded for those more accurate perceptions, so after thousands of generations we can be quite confident that we’re the offspring of those who saw accurately, and so we see accurately. That sounds very plausible. But I think it is utterly false. It misunderstands the fundamental fact about evolution, which is that it’s about fitness functions — mathematical functions that describe how well a given strategy achieves the goals of survival and reproduction. The mathematical physicist Chetan Prakash proved a theorem that I devised that says: According to evolution by natural selection, an organism that sees reality as it is will never be more fit than an organism of equal complexity that sees none of reality but is just tuned to fitness. Never.
-- Donald Hoffman

Today, I think "evolutionary debunking arguments" against realism is fashionable for anti-realists. If Daniel Dennet is right about Darwin, ("If Nietzsche is the father of existentialism, then perhaps Darwin deserves the title of grandfather.") then Darwin is really important figure in the history of anti-realism.

Replies from: TAG
comment by TAG · 2023-05-18T14:24:01.614Z · LW(p) · GW(p)

Hoffman's book has been reviewed as part of Scott's review competition. https://docs.google.com/document/d/1AtGIIv371v0Yu35eNsIxJr67dw4SHOiGdKrqmoKt2hg/edit